Bοήθεια/Απορίες στη Φυσική Προσανατολισμού

djimmakos

Διάσημο μέλος

Ο Μήτσος αυτή τη στιγμή δεν είναι συνδεδεμένος. Είναι 30 ετών και Μαθητής Β' λυκείου. Έχει γράψει 2,790 μηνύματα.
Τον Οκτωβριο στο φροντιστηριο σου ειχανε τελειωσει την υλη ?

Ξεκινάνε από τη Β' την ύλη της Γ'.

Εγώ τώρα κάνω σαν πρώτο πράγμα τις κρούσεις
 

Σημείωση: Το μήνυμα αυτό γράφτηκε 14 χρόνια πριν. Ο συντάκτης του πιθανόν να έχει αλλάξει απόψεις έκτοτε.

mixalis50

Νεοφερμένος

Ο mixalis αυτή τη στιγμή δεν είναι συνδεδεμένος. Είναι 33 ετών και Φοιτητής. Έχει γράψει 16 μηνύματα.
αστεια θεματα
 

Σημείωση: Το μήνυμα αυτό γράφτηκε 14 χρόνια πριν. Ο συντάκτης του πιθανόν να έχει αλλάξει απόψεις έκτοτε.

g!orgos

Εκκολαπτόμενο μέλος

Ο Γιώργος αυτή τη στιγμή δεν είναι συνδεδεμένος. Είναι Φοιτητής και μας γράφει απο Άργος (Αργολίδα). Έχει γράψει 218 μηνύματα.
Δίσκος Δ μάζας Μ=3kg , ισορροπεί συνδεδεμένος στο ένα άκρο κατακόρυφου ιδανικού ελατηρίου σταθεράς k=100 N/m το άλλο άκρο του οποίου είναι στερεωμένο σε ακλόνητο σημείο οροφής. Τη χρονική στιγμή
τοποθετούμε πάνω στο δίσκο σώμα Σ μάζας m=1kg.
α. Ν.δ.ο το σύστημα σώμα -δίσκος-ελατήριο εκτελεί α.α.τ
β. Να υπολογίσετε : 1. το πλάτος Α της ταλάντωσης
2. τη συχνότητα της ταλάντωσης
γ. Να γράψετε τις εξισώσεις ,σε συνάρτηση με την απομάκρυνση ψ του σώματος Σ απο τη Θ.Ι του συστήματος , για
1.τη δύναμη που ασκεί το σώμα Σ στο δίσκο
2. τη δύναμη του ελατηρίου
3. τη κινητική ενέργεια ταλάντωσης του συστήματος

g=10m/s^2 Να θεωρήσετε ως θετική φορά , την φορά προς τα πάνω.
 

Σημείωση: Το μήνυμα αυτό γράφτηκε 14 χρόνια πριν. Ο συντάκτης του πιθανόν να έχει αλλάξει απόψεις έκτοτε.

amalfi

Δραστήριο μέλος

Ο amalfi αυτή τη στιγμή δεν είναι συνδεδεμένος. Είναι 40 ετών και Καθηγητής. Έχει γράψει 458 μηνύματα.
ενα ερωτημα-προκληση για οσους αγαπουν τη φυσικη (και τα μαθηματικα)


------------------------------------------------------------------------------------------------
υλικο σημειο (μπαλιτσα :)) κινειται σε λειο εδαφος και κατευθυνεται προς την κορυφη ενος λοφου.

η αρχικη κινητικη ενεργεια ειναι ακριβως ιση με τη δυναμικη [βαρυτητας] στο ψηλοτερο σημειο.

(η αρχικη δυναμικη ειναι μηδεν)

στο υψηλοτερο σημειο αν το τοποθετουσαμε [ακινητο] θα ισορροπουσε (για παντα)

μπορειτε να προβλεψετε τι θα συμβει?

(θα περασει απ την αλλη μερια του λοφου? θα μεινει στην κορυφη? θα επιστρεψει πισω? κλπ)


(οσοι το βρουν ας μη βιαστουν να απαντησουν για να το παλεψουν και οι υπολοιποι)

(το προβλημα ειναι δυσκολο, αλλα κρυβει πολλη κατανοηση και ουσια)

:bye:


[μπορειτε να θεωρησετε για ευκολια οτι ο λοφος ειναι δυο διαστασεων (οπως η κατακορυφη τομη μιας καμπανας) ]
 

Σημείωση: Το μήνυμα αυτό γράφτηκε 14 χρόνια πριν. Ο συντάκτης του πιθανόν να έχει αλλάξει απόψεις έκτοτε.

hearts_alive

Νεοφερμένος

Ο hearts_alive αυτή τη στιγμή δεν είναι συνδεδεμένος. Είναι Απόφοιτος. Έχει γράψει 78 μηνύματα.
Αρκετά εύκολα τα θέματα, ειδικά τα θέματα θεωρίας, άρα δεν πιστεύω να συνάντησε δυσκολίες ο οποιοσδήποτε που'χε προετοιμαστεί όπως έπρεπε.

Σαφώς ευκολότερα απ'του Μαΐου...
 

Σημείωση: Το μήνυμα αυτό γράφτηκε 14 χρόνια πριν. Ο συντάκτης του πιθανόν να έχει αλλάξει απόψεις έκτοτε.

crisdirk41

Νεοφερμένος

Ο Χρηστος αυτή τη στιγμή δεν είναι συνδεδεμένος. Είναι Φοιτητής. Έχει γράψει 32 μηνύματα.
Γιατί δεν πας στο σχολείο σου να ζητήσεις ένα από τα περσινά βιβλία; Πάντα έχουν.... κ στην ανάγκη πες τους "ο τάδε είμαι το θέλω τώρα μην μου μοιράσετε το σεπτέμβρη" :no1:

Τώρα για βοήθημα με λύσεις του σχολικού μέσα δεν γνωρίζω αν υπάρχει...


Αχ ρε Ελλάδα με την δωρεάν παιδεία σου... όλοι τρέξαμε,τρέχουμε και θα τρέχουμε στα βοηθητικά κ στα φροντιστήρια :P


πηγα στο σχολειο και λενε δε μου δινουν γιατι απαγορευται!!!!!!!!!!!!!!!!!!!! ελεος δηλαδη.... εχω ψαξει στο γκοογκλ αλλα δεν εχω βρει τιποτα... γιαυτο σας ρωταω αν υπαρχουν πουθενα οι λυσεις...
 

Σημείωση: Το μήνυμα αυτό γράφτηκε 14 χρόνια πριν. Ο συντάκτης του πιθανόν να έχει αλλάξει απόψεις έκτοτε.

coheNakatos

Δραστήριο μέλος

Ο Βαγγελης αυτή τη στιγμή δεν είναι συνδεδεμένος. Είναι Μαθητής Γ' λυκείου και μας γράφει απο Βύρωνας (Αττική). Έχει γράψει 662 μηνύματα.
Κουραστικα να προσπαθω στο λατεχ στο 1ο πηρα στην Θ.Ι ΣF=0 (1) πηρα και ΣF=W-Fελ' (για τυχαια θεση) (2) και βγηκε ΣF=-kx Αρα ειναι της μορφης ΣF=-Dx Αρα κανει Α.Α.Τ με D=K. Μετα Δλ=Α Με τα 2 σωματα αφου κανει μονο του Α.Α.Τ χωρις καμμια παρεμβαση .Ε μετα ολα ειναι ευκολα (σορρυ απλα δεν μπορω να συνεχισω εχω γραψει κανα μισαωρο στο λατεχ και απετυχα ):lol:
 

Σημείωση: Το μήνυμα αυτό γράφτηκε 14 χρόνια πριν. Ο συντάκτης του πιθανόν να έχει αλλάξει απόψεις έκτοτε.

fractal-gr

Νεοφερμένος

Ο fractal-gr αυτή τη στιγμή δεν είναι συνδεδεμένος. Είναι 54 ετών και Καθηγητής. Έχει γράψει 21 μηνύματα.
Μια διευκρινιστική ερώτηση πρώτα.

Τι σχέση έχει η διατήρηση της μηχανικής ενέργειας (όπως προκύπτει από την εκφώνηση) με τη περίπτωση ισορροπίας του σώματος στην κορυφή του λόφου;

Υ.Γ. Αν "συλλαμβάνω" το πνεύμα της άσκησης, θα μπορούσε να δοθεί και σε παραλλαγή, με τη μπαλίτσα να κατεβαίνει από την πλαγιά του λόφου κατευθυνόμενη προς τους πρόποδες, όπου εκεί συναντά μια κοιλάδα (μεταξύ 2 λόφων)
 

Σημείωση: Το μήνυμα αυτό γράφτηκε 14 χρόνια πριν. Ο συντάκτης του πιθανόν να έχει αλλάξει απόψεις έκτοτε.

amalfi

Δραστήριο μέλος

Ο amalfi αυτή τη στιγμή δεν είναι συνδεδεμένος. Είναι 40 ετών και Καθηγητής. Έχει γράψει 458 μηνύματα.
Τι σχέση έχει η διατήρηση της μηχανικής ενέργειας (όπως προκύπτει από την εκφώνηση) με τη περίπτωση ισορροπίας του σώματος στην κορυφή του λόφου;
Καλωσηρθες!

πραγματι, η μηχανικη διατηρειται στο προβλημα μας


το στοιχειο που δινω για την ισορροπια ειναι ενα επιπλεον στοιχειο [ασχετο με την ΑΔΜΕ] .
(ειναι ενα αλλο "πειραμα" που κανουμε)
-- (για το "ιδανικο" προβλημα που εχουμε ειναι επομενο να ισορροπει αν το αφησουμε στην κορυφη αλλα το εδωσα για να μην κολλησουν τα παιδια εδω)



(για την παραλλαγη που λες δε βλεπω κατι προς το παρον/ ας λυσουμε αυτη και μας λες μετα)

:)
 

Σημείωση: Το μήνυμα αυτό γράφτηκε 14 χρόνια πριν. Ο συντάκτης του πιθανόν να έχει αλλάξει απόψεις έκτοτε.

Civilara

Περιβόητο μέλος

Ο Civilara αυτή τη στιγμή δεν είναι συνδεδεμένος. Μας γράφει απο Δανία (Ευρώπη). Έχει γράψει 4,344 μηνύματα.
Το σημείο στην κορυφή είναι σημείο ασταθούς ισορροπίας καθώς η παραμικρή εκτροπή από την ισορροπία του σε αυτή τη θέση θα το οδηγήσει σε καταστάσεις μικρότερης δυναμικής ενέργειας χωρίς δυνατότητα να επανέλθει στην αρχική του κατάσταση. Στη θέση αυτή η δυναμική ενέργεια γίνεται μέγιστη και η κινητική ελάχιστη (μηδέν). Επιπλέον επειδή η θέση στην κορυφή του λόφου είναι σημείο ασταθούς ισορροπίας τότε το σώμα είτε θα επιστρέψει πίσω είτε θα περάσει από την άλλη μεριά του λόφου.
 

Σημείωση: Το μήνυμα αυτό γράφτηκε 14 χρόνια πριν. Ο συντάκτης του πιθανόν να έχει αλλάξει απόψεις έκτοτε.

amalfi

Δραστήριο μέλος

Ο amalfi αυτή τη στιγμή δεν είναι συνδεδεμένος. Είναι 40 ετών και Καθηγητής. Έχει γράψει 458 μηνύματα.
(για να μη γινει παρεξηγηση να τονισω οτι το πλαισιο ειναι η κλασικη νευτωνεια μηχανικη (αυτη του σχολειου)

ειμαστε ας πουμε καπου στα 1800 μ.Χ. )

δε θελω να κρινω ακομα την απαντηση σου.
θα ειναι ωραια να γινει συζητηση και με μαθητες - ας προσπαθησουν να την κρινουν τα παιδια!
 

Σημείωση: Το μήνυμα αυτό γράφτηκε 14 χρόνια πριν. Ο συντάκτης του πιθανόν να έχει αλλάξει απόψεις έκτοτε.

_jo_

Νεοφερμένος

Η _jo_ αυτή τη στιγμή δεν είναι συνδεδεμένη. Είναι 34 ετών και Μαθητής Β' λυκείου. Έχει γράψει 3 μηνύματα.
έχω κάτι ασκήσεις στη φυσική(εννοείται) κ' μου βγαίνουν στο περίπου!!προσφέρεται κανείς για βοήθεια στην ΑΑΤ ??
 

Σημείωση: Το μήνυμα αυτό γράφτηκε 14 χρόνια πριν. Ο συντάκτης του πιθανόν να έχει αλλάξει απόψεις έκτοτε.

coheNakatos

Δραστήριο μέλος

Ο Βαγγελης αυτή τη στιγμή δεν είναι συνδεδεμένος. Είναι Μαθητής Γ' λυκείου και μας γράφει απο Βύρωνας (Αττική). Έχει γράψει 662 μηνύματα.
έχω κάτι ασκήσεις στη φυσική(εννοείται) κ' μου βγαίνουν στο περίπου!!προσφέρεται κανείς για βοήθεια στην ΑΑΤ ??

Φυσικα αλλωστε γιαυτο ανοιξα το "τοπικ" μπορεις να τις βαλεις
 

Σημείωση: Το μήνυμα αυτό γράφτηκε 14 χρόνια πριν. Ο συντάκτης του πιθανόν να έχει αλλάξει απόψεις έκτοτε.

Rania.

Πολύ δραστήριο μέλος

Η Rania. αυτή τη στιγμή δεν είναι συνδεδεμένη. Είναι 31 ετών και Φοιτήτρια. Έχει γράψει 1,778 μηνύματα.
Εγω αν μπορω. Αν ειναι απλες δηλαδη. :p

Και οποιος μπορει να μου πει αν στην 5.42 σελ 180 του σχολικου βιβλιου στο β) βγαινει οντως 95% γιατι εγω μονο 90% βρισκω και καπου τα κανω σκατα.
 

Σημείωση: Το μήνυμα αυτό γράφτηκε 14 χρόνια πριν. Ο συντάκτης του πιθανόν να έχει αλλάξει απόψεις έκτοτε.

Black_Butterfly

Πολύ δραστήριο μέλος

Η Αλεξάνδρα αυτή τη στιγμή δεν είναι συνδεδεμένη. Είναι Φοιτήτρια και μας γράφει απο Ηράκλειο (Κρήτη). Έχει γράψει 1,327 μηνύματα.
Και οποιος μπορει να μου πει αν στην 5.42 σελ 180 του σχολικου βιβλιου στο β) βγαινει οντως 95% γιατι εγω μονο 90% βρισκω και καπου τα κανω σκατα.


Ναι 95% βγαίνει


Π(%) = Εαπωλ/ Κολ(πριν)= (Κολ(πριν) - Κολ(μετα)) / Κολπριν = 1/2 mu^2 - 1/2 (m +Μ)V^2 / (1/2 mu^2) = ((0,05 *200^2 - 100) / (0,05 *200^2)) * 100 = 95%
 

Σημείωση: Το μήνυμα αυτό γράφτηκε 14 χρόνια πριν. Ο συντάκτης του πιθανόν να έχει αλλάξει απόψεις έκτοτε.

χρηστοσ17

Νεοφερμένος

Ο χρηστοσ17 αυτή τη στιγμή δεν είναι συνδεδεμένος. Είναι 31 ετών και Μαθητής Γ' λυκείου. Έχει γράψει 90 μηνύματα.
εχουμε ενα σωμα(Σ1) που ειναι ενωμενω με ενα οριζοντιο ελατηριο στου οποιο,το αλλο ακρο ειναι ακλονητο,κανει α.α.τ.και συγκρουεται κατακορυφα και πλαστικα στη θι του Σ1 με ενα αλλο σωμα (Σ2) με αρχικη ταχυτητα (υ1)
ποιο απο τα παρακατω θα αλλαξει και γιατι?
α)η σταθερα κ?
β)το πλατος τησ ταλαντωσεισ?
γ)Fελmax θα αλλαξει?
(οποιοσ ξερει παρακαλω ασ απαντησει)
 

Σημείωση: Το μήνυμα αυτό γράφτηκε 14 χρόνια πριν. Ο συντάκτης του πιθανόν να έχει αλλάξει απόψεις έκτοτε.

χρηστοσ17

Νεοφερμένος

Ο χρηστοσ17 αυτή τη στιγμή δεν είναι συνδεδεμένος. Είναι 31 ετών και Μαθητής Γ' λυκείου. Έχει γράψει 90 μηνύματα.
εχουμε ενα σωμα(Σ1) που ειναι ενωμενω με ενα οριζοντιο ελατηριο στου οποιο,το αλλο ακρο ειναι ακλονητο,κανει α.α.τ.και συγκρουεται κατακορυφα και πλαστικα στη θι του Σ1 με ενα αλλο σωμα (Σ2) με αρχικη ταχυτητα (υ1)
ποιο απο τα παρακατω θα αλλαξει και γιατι?
α)η σταθερα κ?
β)το πλατος τησ ταλαντωσεισ?
γ)Fελmax θα αλλαξει?
(οποιοσ ξερει παρακαλω ασ απαντησει)
 

Σημείωση: Το μήνυμα αυτό γράφτηκε 14 χρόνια πριν. Ο συντάκτης του πιθανόν να έχει αλλάξει απόψεις έκτοτε.

djimmakos

Διάσημο μέλος

Ο Μήτσος αυτή τη στιγμή δεν είναι συνδεδεμένος. Είναι 30 ετών και Μαθητής Β' λυκείου. Έχει γράψει 2,790 μηνύματα.
Yπάρχει ήδη θέμα για τις ασκήσεις φυσικής και απ' ό,τι βλέπω έχεις δημοσιεύσει και εκεί την άσκηση. ( https://ischool.e-steki.gr/showthread.php?t= )
Περίμενε και θα σου απαντήσουν εκεί

Δεν είναι ανάγκη να ανοίγεις συνέχεια καινούργια θέματα όταν υπάρχουν ίδια.

:lock:
 

Σημείωση: Το μήνυμα αυτό γράφτηκε 14 χρόνια πριν. Ο συντάκτης του πιθανόν να έχει αλλάξει απόψεις έκτοτε.

coheNakatos

Δραστήριο μέλος

Ο Βαγγελης αυτή τη στιγμή δεν είναι συνδεδεμένος. Είναι Μαθητής Γ' λυκείου και μας γράφει απο Βύρωνας (Αττική). Έχει γράψει 662 μηνύματα.
Λοιπον , για το 1ο σιγουρα δεν εχουμε αλλαγη του Κ αφου εξαρταται μονο απο τα φυσικα χαρακτιριστικα του (υλικο κλπ.) για το πλατος σιγουρα θα εχουμε αλλαγη , και αυτο γιατι θα αλλαξει η ολικη μαζα (Μ(Σ1)+m(Σ2))
Α.Δ.Ο : Μετα αφου εχουμε την V Που θα ειναι και Umax Για την ταλαντωση ειναι : . Τελος , προφανως αφου αλλαξε το πλατος της ταλαντωσης απο τον τυπο θα εχουμε μεταβολη . Αυτα ...
 

Σημείωση: Το μήνυμα αυτό γράφτηκε 14 χρόνια πριν. Ο συντάκτης του πιθανόν να έχει αλλάξει απόψεις έκτοτε.

χρηστοσ17

Νεοφερμένος

Ο χρηστοσ17 αυτή τη στιγμή δεν είναι συνδεδεμένος. Είναι 31 ετών και Μαθητής Γ' λυκείου. Έχει γράψει 90 μηνύματα.
το σωμα πεφτει ετσι


........................__
........................l__ls2
..........................lu1
........................___
_______________l__l_S1_________________
..........................θι
-----------------------------------------
περνεισ ΑΔΟ και στουσ δυο αξονεσ:( και στον χχ' και στον ΥΥ' κατι που δεν ξερω αν ισχυει...
 

Σημείωση: Το μήνυμα αυτό γράφτηκε 14 χρόνια πριν. Ο συντάκτης του πιθανόν να έχει αλλάξει απόψεις έκτοτε.

Χρήστες Βρείτε παρόμοια

Top